(New page: a) <math>|h(x)| = \left|\int{f(x-y)g(y)dy}\right| \le \int{|f(x-y)g(y)|dy} \le \left(\int{|f(x-y)|^p dy}\right)^{1/p}\left(\int{|g(y)|^q dy}\right)^{1/q}</math> by Holder's Inequality. Le...)
 
 
Line 6: Line 6:
  
 
Since <math>f \in L^p</math> and <math>g \in L^q</math> we get <math>|h(x)| \le ||f||_p ||g||_q < \infty</math>
 
Since <math>f \in L^p</math> and <math>g \in L^q</math> we get <math>|h(x)| \le ||f||_p ||g||_q < \infty</math>
 
b)
 

Latest revision as of 15:59, 22 July 2008

a) $ |h(x)| = \left|\int{f(x-y)g(y)dy}\right| \le \int{|f(x-y)g(y)|dy} \le \left(\int{|f(x-y)|^p dy}\right)^{1/p}\left(\int{|g(y)|^q dy}\right)^{1/q} $ by Holder's Inequality.

Let $ z = y - x $. Then $ \left(\int{|f(x-y)|^p dy}\right)^{1/p} \left(\int{|g(y)|^q}\right)^{1/q} = \left(\int{|f(-z)|^p dz}\right)^{1/p} ||g||_q = ||f||_p ||g||_q $ because $ \int{|f(-x)|} = \int{|f(x)|} $.

Since $ f \in L^p $ and $ g \in L^q $ we get $ |h(x)| \le ||f||_p ||g||_q < \infty $

Alumni Liaison

Correspondence Chess Grandmaster and Purdue Alumni

Prof. Dan Fleetwood